Difference between revisions of "1965 AHSME Problems/Problem 30"

m (added link)
m (see also box)
Line 21: Line 21:
 
== Solution 2 ==  
 
== Solution 2 ==  
 
It's easy to verify that <math>\angle CDA</math> always equals <math>90^\circ</math>. Since <math>\angle CDF</math> changes depending on the sidelengths of the triangle, we cannot be certain that <math>\angle CDF=45^\circ</math>. Hence our answer is <math>\fbox{B}</math>.
 
It's easy to verify that <math>\angle CDA</math> always equals <math>90^\circ</math>. Since <math>\angle CDF</math> changes depending on the sidelengths of the triangle, we cannot be certain that <math>\angle CDF=45^\circ</math>. Hence our answer is <math>\fbox{B}</math>.
 +
 +
== See Also ==
 +
{{AHSME 40p box|year=1965|num-b=29|num-a=31}}
 +
{{MAA Notice}}

Revision as of 09:05, 19 July 2024

Problem 30

Let $BC$ of right triangle $ABC$ be the diameter of a circle intersecting hypotenuse $AB$ in $D$. At $D$ a tangent is drawn cutting leg $CA$ in $F$. This information is not sufficient to prove that

$\textbf{(A)}\ DF \text{ bisects }CA \qquad  \textbf{(B) }\ DF \text{ bisects }\angle CDA \\ \textbf{(C) }\ DF = FA \qquad  \textbf{(D) }\ \angle A = \angle BCD \qquad  \textbf{(E) }\ \angle CFD = 2\angle A$


Solution 1

We will prove every result except for $\fbox{B}$.

By Thales' Theorem, $\angle CDB=90^\circ$ and so $\angle CDA= 90^\circ$. $FC$ and $FD$ are both tangents to the same circle, and hence equal. Let $\angle CFD=\alpha$. Then $\angle FDC = \frac{180^\circ - \alpha}{2}$, and so $\angle FDA = \frac{\alpha}{2}$. We also have $\angle AFD = 180^\circ - \alpha$, which implies $\angle FAD=\frac{\alpha}{2}$. This means that $CF=DF=FA$, so $DF$ indeed bisects $CA$. We also know that $\angle BCD=90-\frac{180^\circ - \alpha}{2}=\frac{\alpha}{2}$, hence $\angle A = \angle BCD$. And $\angle CFD=2\angle A$ as $\alpha = \frac{\alpha}{2}\times 2$.

Since all of the results except for $B$ are true, our answer is $\fbox{B}$.

Solution 2

It's easy to verify that $\angle CDA$ always equals $90^\circ$. Since $\angle CDF$ changes depending on the sidelengths of the triangle, we cannot be certain that $\angle CDF=45^\circ$. Hence our answer is $\fbox{B}$.

See Also

1965 AHSC (ProblemsAnswer KeyResources)
Preceded by
Problem 29
Followed by
Problem 31
1 2 3 4 5 6 7 8 9 10 11 12 13 14 15 16 17 18 19 20 21 22 23 24 25 26 27 28 29 30 31 32 33 34 35 36 37 38 39 40
All AHSME Problems and Solutions

The problems on this page are copyrighted by the Mathematical Association of America's American Mathematics Competitions. AMC logo.png